∼p∨(∼p∧q)≡∼p∧∼q , prove logical equivalence












0












$begingroup$


Our teacher gave us this equivalence to prove in our final exam but almost all of us said that it was not provable and we did not solve it and now we asked him (The Teacher) and he says that it is provable. I was unable to prove it even by using truth table and he asked us to prove it by using the theorems for logical equivalence.



So now we have our grades at stake so please solve it so that I may know that either I'm wrong or the Teacher is!



The theorems are:




  1. Commutative laws: p∧q ≡ q∧p , p∨q ≡ q∨p


  2. Associative laws: (p∧q)∧r ≡ p∧(q∧r) , (p∨q)∨r ≡ p∨(q∨r)


  3. Distributive laws: p∧(q∨r) ≡ (p∧q)∨(p∧r), p∨(q∧r) ≡ (p∨q)∧(p∨r)


  4. Identity laws: p∧t ≡ p , p∨c ≡ p


  5. Negation laws: p∨∼p ≡ t , p∧∼p ≡ c


  6. Double negative law: ∼(∼p) ≡ p


  7. Idempotent laws: p∧p ≡ p , p∨p ≡ p


  8. Universal bound laws: p∨t≡t ,p∧c≡c


  9. De Morgan’s laws: ∼(p∧q) ≡ ∼p∨∼q ,∼(p∨q) ≡ ∼p∧∼q


  10. Absorption laws: p∨(p∧q) ≡ p ,p∧(p∨q) ≡ p


  11. Negations of t and c: ∼t ≡ c , ∼c ≡ t











share|cite|improve this question











$endgroup$












  • $begingroup$
    Well, thanks a lot to everyone for clarifying this point. in fact, I did the same in my paper. I proved that it was not holding logical equivalence and then proved that it was equivalent to (not)p. But the teacher still says that his question is correct and it holds logical equivalence.
    $endgroup$
    – Waqad Arshad
    Jan 12 at 8:55
















0












$begingroup$


Our teacher gave us this equivalence to prove in our final exam but almost all of us said that it was not provable and we did not solve it and now we asked him (The Teacher) and he says that it is provable. I was unable to prove it even by using truth table and he asked us to prove it by using the theorems for logical equivalence.



So now we have our grades at stake so please solve it so that I may know that either I'm wrong or the Teacher is!



The theorems are:




  1. Commutative laws: p∧q ≡ q∧p , p∨q ≡ q∨p


  2. Associative laws: (p∧q)∧r ≡ p∧(q∧r) , (p∨q)∨r ≡ p∨(q∨r)


  3. Distributive laws: p∧(q∨r) ≡ (p∧q)∨(p∧r), p∨(q∧r) ≡ (p∨q)∧(p∨r)


  4. Identity laws: p∧t ≡ p , p∨c ≡ p


  5. Negation laws: p∨∼p ≡ t , p∧∼p ≡ c


  6. Double negative law: ∼(∼p) ≡ p


  7. Idempotent laws: p∧p ≡ p , p∨p ≡ p


  8. Universal bound laws: p∨t≡t ,p∧c≡c


  9. De Morgan’s laws: ∼(p∧q) ≡ ∼p∨∼q ,∼(p∨q) ≡ ∼p∧∼q


  10. Absorption laws: p∨(p∧q) ≡ p ,p∧(p∨q) ≡ p


  11. Negations of t and c: ∼t ≡ c , ∼c ≡ t











share|cite|improve this question











$endgroup$












  • $begingroup$
    Well, thanks a lot to everyone for clarifying this point. in fact, I did the same in my paper. I proved that it was not holding logical equivalence and then proved that it was equivalent to (not)p. But the teacher still says that his question is correct and it holds logical equivalence.
    $endgroup$
    – Waqad Arshad
    Jan 12 at 8:55














0












0








0


1



$begingroup$


Our teacher gave us this equivalence to prove in our final exam but almost all of us said that it was not provable and we did not solve it and now we asked him (The Teacher) and he says that it is provable. I was unable to prove it even by using truth table and he asked us to prove it by using the theorems for logical equivalence.



So now we have our grades at stake so please solve it so that I may know that either I'm wrong or the Teacher is!



The theorems are:




  1. Commutative laws: p∧q ≡ q∧p , p∨q ≡ q∨p


  2. Associative laws: (p∧q)∧r ≡ p∧(q∧r) , (p∨q)∨r ≡ p∨(q∨r)


  3. Distributive laws: p∧(q∨r) ≡ (p∧q)∨(p∧r), p∨(q∧r) ≡ (p∨q)∧(p∨r)


  4. Identity laws: p∧t ≡ p , p∨c ≡ p


  5. Negation laws: p∨∼p ≡ t , p∧∼p ≡ c


  6. Double negative law: ∼(∼p) ≡ p


  7. Idempotent laws: p∧p ≡ p , p∨p ≡ p


  8. Universal bound laws: p∨t≡t ,p∧c≡c


  9. De Morgan’s laws: ∼(p∧q) ≡ ∼p∨∼q ,∼(p∨q) ≡ ∼p∧∼q


  10. Absorption laws: p∨(p∧q) ≡ p ,p∧(p∨q) ≡ p


  11. Negations of t and c: ∼t ≡ c , ∼c ≡ t











share|cite|improve this question











$endgroup$




Our teacher gave us this equivalence to prove in our final exam but almost all of us said that it was not provable and we did not solve it and now we asked him (The Teacher) and he says that it is provable. I was unable to prove it even by using truth table and he asked us to prove it by using the theorems for logical equivalence.



So now we have our grades at stake so please solve it so that I may know that either I'm wrong or the Teacher is!



The theorems are:




  1. Commutative laws: p∧q ≡ q∧p , p∨q ≡ q∨p


  2. Associative laws: (p∧q)∧r ≡ p∧(q∧r) , (p∨q)∨r ≡ p∨(q∨r)


  3. Distributive laws: p∧(q∨r) ≡ (p∧q)∨(p∧r), p∨(q∧r) ≡ (p∨q)∧(p∨r)


  4. Identity laws: p∧t ≡ p , p∨c ≡ p


  5. Negation laws: p∨∼p ≡ t , p∧∼p ≡ c


  6. Double negative law: ∼(∼p) ≡ p


  7. Idempotent laws: p∧p ≡ p , p∨p ≡ p


  8. Universal bound laws: p∨t≡t ,p∧c≡c


  9. De Morgan’s laws: ∼(p∧q) ≡ ∼p∨∼q ,∼(p∨q) ≡ ∼p∧∼q


  10. Absorption laws: p∨(p∧q) ≡ p ,p∧(p∨q) ≡ p


  11. Negations of t and c: ∼t ≡ c , ∼c ≡ t








discrete-mathematics logic






share|cite|improve this question















share|cite|improve this question













share|cite|improve this question




share|cite|improve this question








edited Jan 12 at 8:24







Waqad Arshad

















asked Jan 12 at 8:19









Waqad ArshadWaqad Arshad

52




52












  • $begingroup$
    Well, thanks a lot to everyone for clarifying this point. in fact, I did the same in my paper. I proved that it was not holding logical equivalence and then proved that it was equivalent to (not)p. But the teacher still says that his question is correct and it holds logical equivalence.
    $endgroup$
    – Waqad Arshad
    Jan 12 at 8:55


















  • $begingroup$
    Well, thanks a lot to everyone for clarifying this point. in fact, I did the same in my paper. I proved that it was not holding logical equivalence and then proved that it was equivalent to (not)p. But the teacher still says that his question is correct and it holds logical equivalence.
    $endgroup$
    – Waqad Arshad
    Jan 12 at 8:55
















$begingroup$
Well, thanks a lot to everyone for clarifying this point. in fact, I did the same in my paper. I proved that it was not holding logical equivalence and then proved that it was equivalent to (not)p. But the teacher still says that his question is correct and it holds logical equivalence.
$endgroup$
– Waqad Arshad
Jan 12 at 8:55




$begingroup$
Well, thanks a lot to everyone for clarifying this point. in fact, I did the same in my paper. I proved that it was not holding logical equivalence and then proved that it was equivalent to (not)p. But the teacher still says that his question is correct and it holds logical equivalence.
$endgroup$
– Waqad Arshad
Jan 12 at 8:55










4 Answers
4






active

oldest

votes


















3












$begingroup$

I'd say you're right. Using 10, $∼p∨(∼p∧q)≡∼p$, but $∼p not equiv ∼p∧q.$






share|cite|improve this answer









$endgroup$





















    2












    $begingroup$

    If $p$ is false, the left side is true since it's an "or" with one term true (since $p$ false, (not p) is true). But suppose at the same time that $q$ is true. Then the right is false, since it's an "and" with one term false (since $q$ is true, (not q) is false). So it isn't an equivalence.






    share|cite|improve this answer









    $endgroup$





















      2












      $begingroup$

      If $p$ is false and $q$ is true then $neg pwedgeneg q$ is false and $neg pvee(neg pwedge q)$ is true.



      So there no logical equivalence.






      share|cite|improve this answer









      $endgroup$





















        2












        $begingroup$

        Hint



        Yes you are right. This is not an equivalence. In fact $$∼p∨(∼p∧q)≡∼p$$using a truth table.






        share|cite|improve this answer









        $endgroup$













          Your Answer





          StackExchange.ifUsing("editor", function () {
          return StackExchange.using("mathjaxEditing", function () {
          StackExchange.MarkdownEditor.creationCallbacks.add(function (editor, postfix) {
          StackExchange.mathjaxEditing.prepareWmdForMathJax(editor, postfix, [["$", "$"], ["\\(","\\)"]]);
          });
          });
          }, "mathjax-editing");

          StackExchange.ready(function() {
          var channelOptions = {
          tags: "".split(" "),
          id: "69"
          };
          initTagRenderer("".split(" "), "".split(" "), channelOptions);

          StackExchange.using("externalEditor", function() {
          // Have to fire editor after snippets, if snippets enabled
          if (StackExchange.settings.snippets.snippetsEnabled) {
          StackExchange.using("snippets", function() {
          createEditor();
          });
          }
          else {
          createEditor();
          }
          });

          function createEditor() {
          StackExchange.prepareEditor({
          heartbeatType: 'answer',
          autoActivateHeartbeat: false,
          convertImagesToLinks: true,
          noModals: true,
          showLowRepImageUploadWarning: true,
          reputationToPostImages: 10,
          bindNavPrevention: true,
          postfix: "",
          imageUploader: {
          brandingHtml: "Powered by u003ca class="icon-imgur-white" href="https://imgur.com/"u003eu003c/au003e",
          contentPolicyHtml: "User contributions licensed under u003ca href="https://creativecommons.org/licenses/by-sa/3.0/"u003ecc by-sa 3.0 with attribution requiredu003c/au003e u003ca href="https://stackoverflow.com/legal/content-policy"u003e(content policy)u003c/au003e",
          allowUrls: true
          },
          noCode: true, onDemand: true,
          discardSelector: ".discard-answer"
          ,immediatelyShowMarkdownHelp:true
          });


          }
          });














          draft saved

          draft discarded


















          StackExchange.ready(
          function () {
          StackExchange.openid.initPostLogin('.new-post-login', 'https%3a%2f%2fmath.stackexchange.com%2fquestions%2f3070694%2f%25e2%2588%25bcp%25e2%2588%25a8%25e2%2588%25bcp%25e2%2588%25a7q%25e2%2589%25a1%25e2%2588%25bcp%25e2%2588%25a7%25e2%2588%25bcq-prove-logical-equivalence%23new-answer', 'question_page');
          }
          );

          Post as a guest















          Required, but never shown

























          4 Answers
          4






          active

          oldest

          votes








          4 Answers
          4






          active

          oldest

          votes









          active

          oldest

          votes






          active

          oldest

          votes









          3












          $begingroup$

          I'd say you're right. Using 10, $∼p∨(∼p∧q)≡∼p$, but $∼p not equiv ∼p∧q.$






          share|cite|improve this answer









          $endgroup$


















            3












            $begingroup$

            I'd say you're right. Using 10, $∼p∨(∼p∧q)≡∼p$, but $∼p not equiv ∼p∧q.$






            share|cite|improve this answer









            $endgroup$
















              3












              3








              3





              $begingroup$

              I'd say you're right. Using 10, $∼p∨(∼p∧q)≡∼p$, but $∼p not equiv ∼p∧q.$






              share|cite|improve this answer









              $endgroup$



              I'd say you're right. Using 10, $∼p∨(∼p∧q)≡∼p$, but $∼p not equiv ∼p∧q.$







              share|cite|improve this answer












              share|cite|improve this answer



              share|cite|improve this answer










              answered Jan 12 at 8:46









              PatricioPatricio

              3257




              3257























                  2












                  $begingroup$

                  If $p$ is false, the left side is true since it's an "or" with one term true (since $p$ false, (not p) is true). But suppose at the same time that $q$ is true. Then the right is false, since it's an "and" with one term false (since $q$ is true, (not q) is false). So it isn't an equivalence.






                  share|cite|improve this answer









                  $endgroup$


















                    2












                    $begingroup$

                    If $p$ is false, the left side is true since it's an "or" with one term true (since $p$ false, (not p) is true). But suppose at the same time that $q$ is true. Then the right is false, since it's an "and" with one term false (since $q$ is true, (not q) is false). So it isn't an equivalence.






                    share|cite|improve this answer









                    $endgroup$
















                      2












                      2








                      2





                      $begingroup$

                      If $p$ is false, the left side is true since it's an "or" with one term true (since $p$ false, (not p) is true). But suppose at the same time that $q$ is true. Then the right is false, since it's an "and" with one term false (since $q$ is true, (not q) is false). So it isn't an equivalence.






                      share|cite|improve this answer









                      $endgroup$



                      If $p$ is false, the left side is true since it's an "or" with one term true (since $p$ false, (not p) is true). But suppose at the same time that $q$ is true. Then the right is false, since it's an "and" with one term false (since $q$ is true, (not q) is false). So it isn't an equivalence.







                      share|cite|improve this answer












                      share|cite|improve this answer



                      share|cite|improve this answer










                      answered Jan 12 at 8:44









                      coffeemathcoffeemath

                      2,8451415




                      2,8451415























                          2












                          $begingroup$

                          If $p$ is false and $q$ is true then $neg pwedgeneg q$ is false and $neg pvee(neg pwedge q)$ is true.



                          So there no logical equivalence.






                          share|cite|improve this answer









                          $endgroup$


















                            2












                            $begingroup$

                            If $p$ is false and $q$ is true then $neg pwedgeneg q$ is false and $neg pvee(neg pwedge q)$ is true.



                            So there no logical equivalence.






                            share|cite|improve this answer









                            $endgroup$
















                              2












                              2








                              2





                              $begingroup$

                              If $p$ is false and $q$ is true then $neg pwedgeneg q$ is false and $neg pvee(neg pwedge q)$ is true.



                              So there no logical equivalence.






                              share|cite|improve this answer









                              $endgroup$



                              If $p$ is false and $q$ is true then $neg pwedgeneg q$ is false and $neg pvee(neg pwedge q)$ is true.



                              So there no logical equivalence.







                              share|cite|improve this answer












                              share|cite|improve this answer



                              share|cite|improve this answer










                              answered Jan 12 at 8:46









                              drhabdrhab

                              101k544130




                              101k544130























                                  2












                                  $begingroup$

                                  Hint



                                  Yes you are right. This is not an equivalence. In fact $$∼p∨(∼p∧q)≡∼p$$using a truth table.






                                  share|cite|improve this answer









                                  $endgroup$


















                                    2












                                    $begingroup$

                                    Hint



                                    Yes you are right. This is not an equivalence. In fact $$∼p∨(∼p∧q)≡∼p$$using a truth table.






                                    share|cite|improve this answer









                                    $endgroup$
















                                      2












                                      2








                                      2





                                      $begingroup$

                                      Hint



                                      Yes you are right. This is not an equivalence. In fact $$∼p∨(∼p∧q)≡∼p$$using a truth table.






                                      share|cite|improve this answer









                                      $endgroup$



                                      Hint



                                      Yes you are right. This is not an equivalence. In fact $$∼p∨(∼p∧q)≡∼p$$using a truth table.







                                      share|cite|improve this answer












                                      share|cite|improve this answer



                                      share|cite|improve this answer










                                      answered Jan 12 at 8:48









                                      Mostafa AyazMostafa Ayaz

                                      15.6k3939




                                      15.6k3939






























                                          draft saved

                                          draft discarded




















































                                          Thanks for contributing an answer to Mathematics Stack Exchange!


                                          • Please be sure to answer the question. Provide details and share your research!

                                          But avoid



                                          • Asking for help, clarification, or responding to other answers.

                                          • Making statements based on opinion; back them up with references or personal experience.


                                          Use MathJax to format equations. MathJax reference.


                                          To learn more, see our tips on writing great answers.




                                          draft saved


                                          draft discarded














                                          StackExchange.ready(
                                          function () {
                                          StackExchange.openid.initPostLogin('.new-post-login', 'https%3a%2f%2fmath.stackexchange.com%2fquestions%2f3070694%2f%25e2%2588%25bcp%25e2%2588%25a8%25e2%2588%25bcp%25e2%2588%25a7q%25e2%2589%25a1%25e2%2588%25bcp%25e2%2588%25a7%25e2%2588%25bcq-prove-logical-equivalence%23new-answer', 'question_page');
                                          }
                                          );

                                          Post as a guest















                                          Required, but never shown





















































                                          Required, but never shown














                                          Required, but never shown












                                          Required, but never shown







                                          Required, but never shown

































                                          Required, but never shown














                                          Required, but never shown












                                          Required, but never shown







                                          Required, but never shown







                                          Popular posts from this blog

                                          Can a sorcerer learn a 5th-level spell early by creating spell slots using the Font of Magic feature?

                                          Does disintegrating a polymorphed enemy still kill it after the 2018 errata?

                                          A Topological Invariant for $pi_3(U(n))$